Find out if it's linear transformation

Click For Summary
A linear transformation g from R² to R² cannot exist under the given conditions because the two vectors acted upon by g do not satisfy the necessary linearity properties. Specifically, when attempting to scale the second vector by -4 to match the first, the resulting output does not correspond correctly, as (0, -8) does not equal (5, -4). This discrepancy confirms that the transformation fails to meet the requirements of linearity. Therefore, the conclusion is that such a linear transformation does not exist.
Kernul
Messages
211
Reaction score
7

Homework Statement


Does a linear transformation ##g : \mathbb{R}^2 \rightarrow \mathbb{R}^2## so that ##g((2, -3)) = (5, -4)## and ##g((-\frac{1}{2}, \frac{3}{4})) = (0, 2)## exist?

Homework Equations

The Attempt at a Solution


For a linear transformation to exist we need to know if those two satisfy the following conditions:
##g(\vec v_1 + \vec v_2) = g(\vec v_1) + g(\vec v_2)## for any ##\vec v_1, \vec v_2## in ##\mathbb{R}^2##
##g(\alpha \vec v) = \alpha g(\vec v)## for any scalar ##\alpha##
So I thought I should do like this:
I sum the two vectors ##(2, -3)## and ##(-\frac{1}{2}, \frac{3}{4})## like this ##g((2, -3) + (-\frac{1}{2}, \frac{3}{4}))## and put it equal to the sum of ##(5, -4)## and ##(0, 2)##. I will then have something like this:
##g((2, -3) + (-\frac{1}{2}, \frac{3}{4})) = (5, -4) + (0, 2)##
##g((\frac{3}{2}, -\frac{9}{4})) = (5, -2)##
But at this point I actually don't know if it really exist or not. Am I missing something?
 
Physics news on Phys.org
Kernul said:

Homework Statement


Does a linear transformation ##g : \mathbb{R}^2 \rightarrow \mathbb{R}^2## so that ##g((2, -3)) = (5, -4)## and ##g((-\frac{1}{2}, \frac{3}{4})) = (0, 2)## exist?

Am I missing something?

Do you notice anything about the two vectors being acted on by ##g##?
 
  • Like
Likes Kernul
PeroK said:
Do you notice anything about the two vectors being acted on by ##g##?
Oh! If I multiply by ##-4## the second vector acted on by ##g##, I'll get the first vector! But I have to multiply the vector that is is equal to by ##-4## too. And since ##(0, -8)## is not equal to ##(5, -4)## means that the linear transformation doesn't exist, right?
 
Kernul said:
Oh! If I multiply by ##-4## the second vector acted on by ##g##, I'll get the first vector! But I have to multiply the vector that is is equal to by ##-4## too. And since ##(0, -8)## is not equal to ##(5, -4)## means that the linear transformation doesn't exist, right?

Exactly.
 
  • Like
Likes Kernul
Question: A clock's minute hand has length 4 and its hour hand has length 3. What is the distance between the tips at the moment when it is increasing most rapidly?(Putnam Exam Question) Answer: Making assumption that both the hands moves at constant angular velocities, the answer is ## \sqrt{7} .## But don't you think this assumption is somewhat doubtful and wrong?

Similar threads

Replies
3
Views
2K
  • · Replies 9 ·
Replies
9
Views
1K
  • · Replies 2 ·
Replies
2
Views
2K
Replies
1
Views
1K
  • · Replies 16 ·
Replies
16
Views
2K
  • · Replies 9 ·
Replies
9
Views
2K
  • · Replies 11 ·
Replies
11
Views
2K
  • · Replies 3 ·
Replies
3
Views
3K
  • · Replies 13 ·
Replies
13
Views
2K
  • · Replies 6 ·
Replies
6
Views
3K